Visita el Chat que está en la parte inferior de la página

jueves, 27 de mayo de 2010

Mexicoon city

Cada vez falta menos para el misterioso trailer D:

sábado, 22 de mayo de 2010

Trailer CC3

Bueno como nadie da con la respuesta al acertijo, he aqui un adelanto del video. Espero que la disfruten, y que no llegue ninguna demanda.



P.D. Si lo quieren descargar aqui esta el link.
* http://www.megaupload.com/?d=EITPW4SV

Y sigan pensando en el acertijo

viernes, 21 de mayo de 2010

Unicameteate solamente hoy


Mucho se rumora de si existe o no en realidad el video de la CC3, bueno pues para los excepticos de su existencia pueden acceder a el si descifran el acertijo, el te dice la direccion de la pagina:


"...Los que andamos parecemos acertijo, tal cual abejas corriendo con jarras andando por, aras roeremos, a quien no mire, ... por cierto la ultima letra sera la A, de asistentes, y con_Jalea"


y obviamente ya saben, esta en blogspot, y tienen 5 dias a partir de hoy, o su existencia sera movida de lugar, a ese tiempo les dare la respuesta.

jueves, 20 de mayo de 2010

miércoles, 19 de mayo de 2010

e y π


Hoy voy a presentarles una bonita integral (doble)
$$\int _0^{\infty }\int _0^{\infty }e^{-\left(x^2+y^2\right)}dxdy$$
Graficamente representa el volumen entre la superficie llamada Campana de Gauss (en 3D) y el plano xy

Como se habrán dado cuenta no podemos integrar esta función en coordenadas cartesianas, asi que haremos un cambio de variable a coordenadas polares (En términos del radio y el ángulo).
Ya sabemos que
$$x=R \text{Cos}[\theta ]$$ ...1
$$y=R \text{Sen}[\theta ]$$ ...2
Pero además se tiene que añadir un factor de correción para ajustar los nuevos diferenciales llamado Jacobiano.
el cuál está definido por

Asi
$$\int \int _Df(x,y)dxdy=\int \int _Tf(R,\theta )|J|dRd\theta $$ ...3
De la ecuación 1
$$\frac{\partial x}{\partial R}=\text{Cos}[\theta ]$$
$$\frac{\partial x}{\partial \theta }=-R \text{Sen}[\theta ]$$
$$\frac{\partial y}{\partial R}=\text{Sen}[\theta ]$$
$$\frac{\partial y}{\partial \theta }=R \text{Cos}[\theta ]$$
Entonces

Como $$R=\sqrt{x^2+y^2}$$ y  $$0\leq x\leq \infty , 0\leq y\leq \infty $$ entonces $$ 0\leq R\leq \infty $$

y como solo estamos en el primer cuadrante $$0\leq \theta \leq \frac{\pi }{2}$$
Por lo tanto
$$\int _0^{\infty }\int _0^{\infty }e^{-\left(x^2+y^2\right)}\text{dxdy}=\int _0^{\frac{\pi }{2}}\int _0^{\infty }\text{Re}^{-R^2}dRd\theta $$
Y ahora esta integral se ve más facil para realizar.
Haciendo un cambio de variable
$$u=-R^2$$
$$du=-2RdR$$
$$dR=-\frac{du}{2R}$$
$$\int _0^{\frac{\pi }{2}}\int _0^{\infty }\text{Re}^{-R^2}dRd\theta =-\frac{1}{2}\int _0^{\frac{\pi }{2} }\int _0^{\infty }e^udud\theta $$

$$\int _0^{\frac{\pi }{2} }\int _0^{\infty }\text{Re}^{-R^2}dRd\theta =-\frac{1}{2}\int_0^{\frac{\pi }{2} } \left[e^{-u}\right]_0^{\infty } \, d\theta $$
$$\int _0^{\frac{\pi }{2} }\int _0^{\infty }\text{Re}^{-R^2}dRd\theta =-\frac{1}{2}\int_0^{\frac{\pi }{2}} \underset{b\to \infty }{\text{Lim}}\left[e^{-b}\right]_0^b \, d\theta $$
$$\int _0^{\frac{\pi }{2} }\int _0^{\infty }\text{Re}^{-R^2}dRd\theta =-\frac{1}{2}\int_0^{\frac{\pi }{2} } \left[\underset{b\to \infty }{\text{Lim}}e^{-b}-1\right] \, d\theta $$
Ya habiamos visto que $$\underset{b\to \infty }{\text{Lim}}e^{-b}=0$$
Entonces $$\int _0^{\frac{\pi }{2} }\int _0^{\infty }\text{Re}^{-R^2}dRd\theta =\frac{1}{2}\int _0^{\frac{\pi }{2}}d\theta $$
$$\int _0^{\frac{\pi }{2} }\int _0^{\infty }\text{Re}^{-R^2}dRd\theta =\frac{1}{2}[\theta ]_0^{\frac{\pi }{2}}$$
$$\int _0^{\frac{\pi }{2} }\int _0^{\infty }\text{Re}^{-R^2}dRd\theta =\frac{1}{2}[\frac{\pi }{2} ]$$
$$\int _0^{\frac{\pi }{2} }\int _0^{\infty }\text{Re}^{-R^2}dRd\theta =\frac{\pi }{4}$$
$$\therefore \int _0^{\infty }\int _0^{\infty }e^{-\left(x^2+y^2\right)}\text{dxdy}=\frac{\pi }{4}$$
Un caso particular cuando las dos variables de integración son iguales
$$\int _0^{\infty }\int _0^{\infty }e^{-\left(\alpha ^2+\alpha ^2\right)}\text{d$\alpha $d$\alpha $}=\int _0^{\infty }e^{-\alpha ^2}\text{d$\alpha $}\int _0^{\infty }e^{-\alpha ^2}\text{d$\alpha $}$$
$$\int _0^{\infty }\int _0^{\infty }e^{-\left(\alpha ^2+\alpha ^2\right)}\text{d$\alpha $d$\alpha $}=\left[\int _0^{\infty }e^{-\alpha ^2}\text{d$\alpha $}\right]{}^2$$
Usando el resultado que obtuvimos
$$\left[\int _0^{\infty }e^{-\alpha ^2}\text{d$\alpha $}\right]{}^2=\frac{\pi }{4}$$
Sacando raiz de ambos lados
$$\int _0^{\infty }e^{-\alpha ^2}\text{d$\alpha $}=\frac{\sqrt{\pi }}{2}$$
Lo que representa el área entre la campana de Gauss (en 2D) y el eje x

domingo, 16 de mayo de 2010

CC3 :3

Dado que nuestro hermano Markovnikov no ha posteado todavia el video oficial de la CC3, he aqui un prologo de lo que proximamente se revelara al mundo







PD: Soy admin soy admin troololololololololo

lunes, 10 de mayo de 2010